หัวข้อ: Inequality Marathon
ดูหนึ่งข้อความ
  #74  
Old 29 สิงหาคม 2007, 20:50
tatari/nightmare's Avatar
tatari/nightmare tatari/nightmare ไม่อยู่ในระบบ
ลมปราณคุ้มครองร่าง
 
วันที่สมัครสมาชิก: 29 กรกฎาคม 2007
ข้อความ: 276
tatari/nightmare is on a distinguished road
Default

อ้างอิง:
ข้อความเดิมเขียนโดยคุณ kanakon View Post
Let $n\geq 3$ be an integer. Let $t_1,t_2,t_3,...t_n$ be posiive real integer such that
$$(n^2+1)>(t_1+t_2+...+t_n)(\frac{1}{t_1}+\frac{1}{t_2}+...+\frac{1}{t_n} )$$
Show that $t_i,t_j,t_k$ are side lenghts of a triangle for all $i,j,k$ with $1\leq i<j<k\leq n$ [IMO 2004]
$(t_1+t_2+...+t_n)(\frac{1}{t_1}+\frac{1}{t_2}+...+\frac{1}{t_n})=n+\sum_{i<j}^{}(\frac{t_i}{t_j}+\frac{t_j}{t_i})
=n+t_1(\frac{1}{t_2}+\frac{1}{t_3})+\frac{1}{t_1}(t_2+t_3)+\sum_{i<j,(i,j)\not\in{(1,2),(1,3)}}^{}(\frac{t_i}{t_j}+\frac{t_j}{t_ i})$
จาก $$\frac{1}{t_2}+\frac{1}{t_3}\geq\frac{2}{\sqrt{t_{2}t_{3}}}$$
$$t_2+t_3\geq 2\sqrt{t_{2}t_{3}}$$ ( AM-GM)
และ $\frac{t_i}{t_j}+\frac{t_j}{t_i}\geq2$ ให้ $a=\frac{t_1}{\sqrt{t_{2}t_{3}}}$ ได้ $a>0$ จากโจทย์ได้
$n^2+1>(t_1+t_2+...+t_n)(\frac{1}{t_1}+\frac{1}{t_2}+...+\frac{1}{t_n})\geq n+\frac{2t_1}{\sqrt{t_{2}t_{3}}}+\frac{2\sqrt{t_{2}t_{3}}}{t_1}+2[\binom{n}{2}-2]=2a+\frac{2}{a}+n^2-4$
$\therefore 2a+\frac{2}{a}-5< 0$ นั่นคือ $2a^2-5a+2<0$ จะได้ $\frac{1}{2}<a<2$
นั่นคือ $\frac{t_1}{\sqrt{t_{2}t_{3}}}=a<2$
$\therefore t_1<2\sqrt{t_{2}t_{3}}$ แต่จาก $2\sqrt{t_{2}t_{3}}\leq t_2+t_3$
ฉะนั้น $t_1<t_2+t_3$
__________________
AL-QAEDA(เอXข้างหน้า!!)!!!!!!!!!!
ถึง บิน ลาเดนจะลาโลกไปแล้ว แต่เรายังมีผู้นำ jihad คนใหม่....อย่าง
อับดุล อาบาเร่ คราลิดทากัน...เราจะใช้รถดูดส้XXเป็นคาร์บอม!!!จงพลีชีพเพื่อผู้นำของเรา!!!!!!!

BOOM!!!!!!!!!!!!!!!!!!!!!

29 สิงหาคม 2007 20:51 : ข้อความนี้ถูกแก้ไขแล้ว 1 ครั้ง, ครั้งล่าสุดโดยคุณ tatari/nightmare
ตอบพร้อมอ้างอิงข้อความนี้